LSAT and Law School Admissions Forum

Get expert LSAT preparation and law school admissions advice from PowerScore Test Preparation.

 Administrator
PowerScore Staff
  • PowerScore Staff
  • Posts: 8919
  • Joined: Feb 02, 2011
|
#40677
Complete Question Explanation
(The complete setup for this game can be found here: lsat/viewtopic.php?t=15486)

The correct answer choice is (D)

In this List question, the rules should be applied in the following order: second, fourth, fifth, third, and first.

Answer choice (A): This answer choice violates the second rule and is therefore incorrect.

Answer choice (B): This answer choice violates the second rule and the fourth rule and is therefore incorrect.

Answer choice (C): This answer choice violates the second rule and is therefore incorrect.

Answer choice (D): This is the correct answer choice.

Answer choice (E): This answer choice violates the third rule and is therefore incorrect.
This question is unusual in that the first three answers all violate the second rule, a very easy rule to apply visually.

Get the most out of your LSAT Prep Plus subscription.

Analyze and track your performance with our Testing and Analytics Package.